Differentiation using product rule?

Click For Summary
The discussion focuses on differentiating the function p(x) = ((x+5)^2)*((x+3)^7) using the product rule. The initial differentiation attempt yielded a correct result but was deemed incorrect due to lack of simplification. Participants suggested factoring out common elements to simplify the expression further. The final simplified form of the derivative is presented as ((x+3)^6)*((9x^2)+(86x)+205) or alternatively as (x+3)^6*(x+5)*(9x+1). The conversation emphasizes the importance of proper algebraic manipulation after differentiation.
steve snash
Messages
50
Reaction score
0

Homework Statement


Differentiate the following function with respect to x,
p(x) = (( x+5 )^2)*(( x+3 )^7)


Homework Equations


well the product rule is,
p(x)=(f)*(g)
p'(x)= (f')*(g)+(g')*(f)
and general differentiation is,
p'(x)=n(f)^(n-1)*n(g)^(n-1)

The Attempt at a Solution


well i used the product rule and got
(2(x+5))*((x+3)^7)+(7(x+3)^6)*((x+5)^2)
but this is said to be wrong how do i simplify it more or what have i done wrong?
 
Physics news on Phys.org
What you did is right. They probably just want you to pull out the common factors to simplify it more.
 
so i could go,
(2*x+10)*(x+3)^7+(7x+21)*(x+5)^2
then
(3x+13)^7+(8x+26)^2
 
steve snash said:
so i could go,
(2*x+10)*(x+3)^7+(7x+21)*(x+5)^2
then
(3x+13)^7+(8x+26)^2

Your algebra is looking pretty seriously awful there. Whoa. Just factor out (x+5)*(x+3)^6 and collect the rest. Try and use only real algebra this time, and not just random symbol rearrangment, ok?
 
so it works out to be,
((x+3)^6)*((9x^2)+(86x)+205)
cheers for the help
 
steve snash said:
so it works out to be,
((x+3)^6)*((9x^2)+(86x)+205)
cheers for the help

That's one version. You could also write it as (x+3)^6*(x+5)*(9x+1). Whatever works, like I said your initial differentiation was correct as well.
 
Question: A clock's minute hand has length 4 and its hour hand has length 3. What is the distance between the tips at the moment when it is increasing most rapidly?(Putnam Exam Question) Answer: Making assumption that both the hands moves at constant angular velocities, the answer is ## \sqrt{7} .## But don't you think this assumption is somewhat doubtful and wrong?

Similar threads

  • · Replies 10 ·
Replies
10
Views
2K
Replies
2
Views
1K
  • · Replies 2 ·
Replies
2
Views
1K
  • · Replies 16 ·
Replies
16
Views
2K
  • · Replies 11 ·
Replies
11
Views
3K
Replies
2
Views
2K
Replies
1
Views
2K
  • · Replies 1 ·
Replies
1
Views
1K
  • · Replies 7 ·
Replies
7
Views
2K
Replies
3
Views
2K